+1 Daumen
313 Aufrufe

Aufgabe:

Sei h : ℝ→ℝ (f.ü.) definiert durch h(t) = \( \frac{1}{2} \)  für t > 0 und h(t) = - \( \frac{1}{2} \) für t < 0. Zeigen Sie, dass h' = δ im distributionellen Sinne gilt, d.h. dass

∫ h(-φ')dt = φ(0) für alle φ ∈ C∞ (ℝ) gilt.


Problem/Ansatz:

Ich komme mit dieser Aufgabe leider gar nicht zurecht und weiß nicht genau was ich machen soll. Ich hab ein wenig rumprobiert und bin auf einen Ansatz gekommen, allerdings bin ich mir unsicher ob dies richtig ist.

Mein Ansatz war folgender:

T'h(φ) = - \( \int\limits_{-\infty}^{\infty} \) h(t)φ'(t)dt = -( \( \int\limits_{0}^{\infty} \) \( \frac{1}{2} \) φ'(t) dt + \( \int\limits_{-\infty}^{0} \)  ( - \( \frac{1}{2} \) φ'(t)dt)  = ?

Ich steh auch ein bisschen auf dem Schlauch was φ(0) genau ist/ sein soll.

Es handelt sich hierbei um eine Mathematik für Physiker 3 Vorlesung.


Lg, Bobette

Avatar von
Tipp, der hoffentlich nicht noch zusätzlich verwirrt. https://de.wikipedia.org/wiki/Delta-Distribution Ansonsten bitte Geduld, bis sich MathePeter oder jemand anders nochmals meldet.

1 Antwort

0 Daumen
 
Beste Antwort

Hallo,

das ist alles richtig - am Anfang sollte nur stehen: \(T'_h(\phi)\), Du brauchst nur zuende schreiben. Distributionen sind ja Abbildungen von \(C^{\infty}(\mathbb{R})\) nach \(\mathbb{R}\), d.h. in Deiner Rechnung ist \(\phi\) beliebig und über \(\phi(0)\) kannn nichts weiter gesagt werden.

Braucht aber auch nicht, denn Dein Ergebnis soll ja sein \(\delta(\phi)=\phi(0)\).

Gruß

Avatar von 13 k

Hallo MathePeter,

vielen Dank für deine schnelle Antwort.

Leider hänge ich an dem Punkt dies zu Ende zu schreiben. Ich wollte das weiter fortführen in dem ich im nächsten Schritt = \( \int\limits_{0}^{\infty} \)  0*φ(t)dt + \( \int\limits_{-\infty}^{0} \)  0*φ(t)dt = \( \int\limits_{ℝ}^{} \) h'(t)*φ(t) schreibe.

Das kommt mir allerdings ein bisschen quatschig vor. Mein Gedanke dahinter war \( \int\limits_{ℝ}^{} \) h'(t)φ(t)dx = - \( \int\limits_{ℝ}^{} \)h(t) φ'(t) dt zu benutzen. Und dachte mir die Ableitung von \( \frac{1}{2} \) = 0.

Ich sehe Deine Probleme nicht, Du kannst doch einfach ausrechnen:

$$\int_0^{\infty} \frac{1}{2} \phi'(t) dt=\frac{1}{2} \left. \phi(t) \right|_0^{\infty}=-\frac{1}{2} \phi(0)$$

Der Beitrag an der oberen Grenze verschwindet ja, weil \(\phi\) kompakten Träger hat.

Vielen dank. Es ist mir tatsächlich dann selbst doch noch aufgefallen. Ich hab mich zu sehr an einem anderen Beispiel entlang gehangelt.


Bist du DER MathePeter?

Falls ja, ich liebe deine Videos


Lg.

Schade, dass ich nicht DER MathePeter bin

Ein anderes Problem?

Stell deine Frage

Willkommen bei der Mathelounge! Stell deine Frage einfach und kostenlos

x
Made by a lovely community